Number Theory 33

(Gabriel Dospinescu)
[Bài toán]: CMR 2^{3^n}+1 có ít nhất n ước nguyên tố dạng 8k+3
                                                      Lời giải
Nhận xét (Tương tự bài Vietnam TST 2004): 2^n+1 không có ước nguyên tố dạng 8k-3,8k-1
Xét n chẵn: Ta có 2^n\equiv -1\pmod p\Rightarrow \left ( \frac{-1}{p} \right )=1\Rightarrow (-1)^{\frac{p-1}{2}}\equiv 1\pmod 4\Rightarrow p\equiv 1\pmod 4 (mâu thuẫn)
Xét n lẻ. Ta có: 2^{n+1}\equiv -2\pmod p\Rightarrow \left ( \frac{-2}{p} \right )=\left ( \frac{-1}{p} \right )\left ( \frac{2}{p} \right )=1
(-1)^{\frac{p-1}{2}}.(-1)^{\frac{p^2-1}{8}}=1. Mặt khác: p\equiv 5;7\pmod 8\Rightarrow (-1)^{\frac{p-1}{2}}.(-1)^{\frac{p^2-1}{8}}=-1 (mâu thuẫn) \rightarrow Q.E.D
Từ nhận xét trên, do đó mọi ước ước nuyên tố của 2^{3^n}+1 đều có dạng 8k+1,8k+3
Ta có: 2^{3^n}+1=(2+1)(2^2-2+1)(2^{2.3}-2^3+1)...(2^{2.3^{n-1}}-2^{3^{n-1}}+1)
Đặt S_i=2^{2.3^i}-2^{3^i}+1\forall i=\overline{1,n-1}
Ta có: S_i\equiv 1-(-1)+1\equiv 3\pmod 9\Rightarrow 3||S_i\forall i=\overline{1,n-1}\qquad (1)
Gọi p|\gcd(S_i,S_j)\forall i,j=\overline{1,n-1}. Ta có: p|S_i|2^{3^{i+1}}+1
Ta có: 2^{3^j}=(2^{3^{i+1}})^{3^{j-i-1}}\equiv (-1)^{3^{j-i-1}}\equiv -1\pmod p
\Rightarrow 0\equiv S_j\equiv 2^{2.3^j}-2^{3^j}+1\equiv 1+1+1\equiv 3\pmod p\Rightarrow p=3\qquad (2)
Từ (1),(2)\Rightarrow \gcd(S_i,S_j)=3\forall i,j=\overline{1,n-1}
Nếu mọi ước nguyên tố của S_i (trừ 3) đều có dạng 8k+1\Rightarrow S_i\equiv 3\pmod 8 (vô lí)
Do đó S_i có ước nguyên tố dạng 8k+3p_i\neq 3 do \gcd(S_i,S_j)=3
\Rightarrow p_i\neq p_i\forall i,j=\overline{1,n-1}
Vậy  2^{3^n}+1 có ít nhất n ước nguyên tố dạng 8k+33,p_1,p_2,...,P_{n-1} \blacksquare

Number Theory 26

(Phát triển từ bài toán Taiwan 1999)
[Bài toán]: Cho b nguyên dương sao cho b+1=q là số nguyên tố. Gọi p là ước nguyên tố lớn nhất của b^y+1,q là ước nguyên tố nhỏ nhất của y. CMR: p\geq q+2.
                                                Lời giải
Phản chứng, nếu p\geq q, gọi \text{ord}_pb=h\qquad (h\in \mathbb{Z^+}) \qquad (1)
ta có: p|b^y+1\Leftrightarrow b^{2y}\equiv 1\pmod p \Longrightarrow h|2y\qquad (2)
Từ (1)\Rightarrow h|p-1\Rightarrow p-1\geq h\Leftrightarrow p\geq h+1;p\leq q\Rightarrow h+1\leq q\Rightarrow h<q\Rightarrow \gcd(y,h)=1\qquad (3)
Từ (2),(3)\Rightarrow h|2\Rightarrow h\in \{1;2\}
• Xét h=1\Rightarrow p|b-1. Ta có: p|b^y+1\Rightarrow p|2\Rightarrow p|2\Rightarrow p=2\Rightarrow b lẻ \Rightarrow q chẵn \Rightarrow q=2 (vô lí)
• Xét h=2\Rightarrow p|(b-1)(b+1)\Leftrightarrow p|q(q-2). Nếu \gcd(q,q-2)>1\Rightarrow q chẵn \Rightarrow q=2 (vô lí) \Longrightarrow \gcd(q,q-2)=1. Nếu p|q-2\Rightarrow p|b-1\Rightarrow q=2 (vô lí) \Longrightarrow p|q\Rightarrow q=pp là ước nguyên tố lớn nhất của b^y+1
\Rightarrow q là ước duy nhất của b^y+1. Đặt b^y+1=q^n với n\in \mathbb{Z^+}
Ta có phương trình: (q-1)^y+1=q^n\Rightarrow (q-1)^{2y}\equiv 1\pmod {q^n}. Đặt \text{ord}_{q^n}q-1=h với h\in \mathbb{Z^+}
\Rightarrow h|2y\text{ord}_q(q-1)=2\Rightarrow \text{ord}_{q^n}q-1=2q^{n-1}\Rightarrow 2q^{n-1}|2y \Leftrightarrow q^{n-1}|y\Rightarrow q|y. Đặt y=kq với k\in \mathbb{Z^+}
Dễ dàng thấy y lẻ, nếu y chẵn \Rightarrow q|2\Rightarrow q=2 (vô lí)
\Longrightarrow k lẻ. Ta thấy q^n=(q-1)^y+1=[(q-1)^q]^k+1=[(q-1)^q+1].A\vdots (q-1)^q+1q\in \mathbb{P}\Rightarrow \exists m\in \mathbb{Z^+}:(q-1)^q+1=q^m
q\geq 5\Rightarrow (q-1)^q+1\geq (q-1)^5+1\geq q^3\Rightarrow q^m\geq q^3\Leftrightarrow m\geq 3
Ta có: (q-1)^q+1=q^q-\text{C}_p^1q^{q-1}+...+q^2\equiv q^2\pmod {q^3}q^m\equiv 0\pmod {q^3}\forall m\geq 3\Rightarrow VT\neq NP (mâu thuẫn). Vậy giả thiết phản chứng sai \Rightarrow p>qp,q nguyên tố \Rightarrow p\geq q+2
Kết luận: Vậy ta có Q.E.D \blacksquare

Number Theory 25

[Bài toán]: Cho a,b,c,d\in \mathbb{Z^+} thỏa mãn a^2+b^2\mid ac+bd. CMR: \gcd(a^2+b^2,c^2+d^2)>1
                                                               Lời giải
Ta có: (a^2+b^2)(c^2+d^2)=(ac+bd)^2+(ad-bc)^2
a^2+b^2\mid ac+bd\Rightarrow a^2+b^2\mid (ad-bc)^2
Giả sử \gcd(a^2+b^2,c^2+d^2)=1\qquad (*)
\blacktriangleright Xét a^2+b^2 không là số chính phương \Rightarrow \exists p có số mũ lớn nhất là số lẻ, với p là ước nguyên tố của a^2+b^2, gọi số mũ đó là 2k+1 với k\in \mathbb{N}
Ta có: p^{2k+1}\mid a^2+b^2\Rightarrow p^{2k+1}\mid (ad-bc)^2\Rightarrow p^{2k+2}\mid (ad-bc)^2p^{2k+1}\mid ac+bd\Rightarrow p^{2k+2}\mid p^{4k+2}\mid (ac+bd)^2
\Longrightarrow p^{2k+2}\mid (ad-bc)^2+(ac+bd)^2\Rightarrow p^{2k+2}\mid (a^2+b^2)(c^2+d^2) mà từ (*)\Rightarrow \gcd(p^{2k+2},c^2+d^2)=1\Rightarrow p^{2k+2}\mid a^2+b^2\rightarrow \text{False}
\blacktriangleright Xét a^2+b^2 là số chính phương. Đặt a^2+b^2=t^2 với t\in \mathbb{Z^+}
Ta có: t^2\mid (ad-bc)^2\Rightarrow t\mid ad-bc\Rightarrow ad-bc=ty với y\in \mathbb{Z}
Lại có: t^2\mid ac+bd\Rightarrow ac+bd=t^2x với x\in \mathbb{Z^+}
\Longrightarrow \begin{cases}a^2d^2-b^2c^2=ty(ad+bc)\\a^2c^2-b^2d^2=t^2x(ac-bd)\end{cases} \Rightarrow (a^2-b^2)(c^2+d^2)=t[y(ad+bc)+x(ac-bd)]\Rightarrow t\mid (a^2-b^2)(c^2+d^2)(*)\Rightarrow \gcd(t,c^2+d^2)=1\Rightarrow t\mid a^2-b^2 \Rightarrow \left | a^2-b^2 \right |\geq t
+, Nếu a^2-b^2\geq t, b^2-a^2\geq t\rightarrow \text{False}
+, Nếu a^2=b^2\Rightarrow a=b\Rightarrow 2a^2=t^2\rightarrow \text{False}
Kết luận: Vậy giả sử sai, ta có Q.E.D \blacksquare 

Number Theory 24

(IMO 2008 Shortlist)
[Bài toán]: Cho p\in \mathbb{P}, n\in \mathbb{Z^+},a,b,c\in \mathbb{Z} thỏa mãn a^n+pb=b^n+pc=c^n+pa.
CMR: a=b=c
                                                           Lời Giải
Giả sử a,b,c đôi một khác nhau.
Ta có: a^n+pb=b^n+pc=c^n+pa\Leftrightarrow \begin{cases}a^n-b^n=p(c-b)\\b^n-c^n=p(a-c)\\c^n-a^n=p(b-c)\end{cases}
\Rightarrow -p^3=\displaystyle{\frac{a^n-b^n}{a-b}.\frac{b^n-c^n}{b-c}.\frac{c^n-a^n}{c-a}}\qquad (1)
Xét n lẻ \Rightarrow a^n-b^n,a-b cùng dấu \Rightarrow VP_{(1)}>0VT_{(1)}<0 (vô lí)
Xét n chẵn:
\blacktriangleright Xét p lẻ, p\geq 3. Ta có: -p^3=\prod (a^{n-1}+a^{n-2}b+...+ab^{n-2}+b^{n-1})
Ta thấy a^{n-1}+a^{n-2}b+...+ab^{n-2}+b^{n-1} (2) lẻ mà số các số trong tổng (2) chẵn do n chẵn \Rightarrow a,b khác dấu \Rightarrow a-b\equiv 1\pmod 2
Tương tự \Rightarrow 3\equiv (a-b)+(b-c)+(c-a)\equiv 0\pmod 2 (vô lí)
\blacktriangleright Xét p chẵn \Rightarrow p=2. Đặt n=2k với k\in \mathbb{Z^+},n\geq 2.
(*) Nếu trong ba số có một số bằng 0. Giả sử a=0.
Ta có: \begin{cases}b^n+2c=c^n\\2b=c^n\end{cases}\Rightarrow c^{n^2}+2^{n+1}c=2^nc^n\Leftrightarrow (c^{n-1})^{n+1}-4+4=2^n(c^{n-1}-2)
\Leftrightarrow (c^{n-1}-2)A+4=2^n(c^{n-1}-2)\Leftrightarrow (c^{n-1}-2)(2^n-A)=4 (dễ dàng thấy vô lí)
(*) Nếu cả ba số đều khác 0\Rightarrow \left | a \right |,\left | b \right |,\left | c \right |\geq 1
+, Với k\geq 2. Ta có: \displaystyle{8=\left | \prod \frac{a^n-b^n}{a-b} \right |=\left | \prod (a+b)\frac{a^{2k}-b^{2k}}{a^2-b^2} \right |}
=\left | (a+b)(b+c)(c+a) \right |\left | \prod (a^{2k-2}+...+b^{2k-2}) \right |\geq 8\left | (a+b)(b+c)(c+a) \right |
\Leftrightarrow \left | (a+b)(b+c)(c+a) \right |\leq 1\rightarrow \text{False}
+, Với k=1\Rightarrow (a+b)(b+c)(c+a)=-8. Từ giả thiết, dễ dàng thấy a,b,c cùng tính chẵn lẻ \Rightarrow \left | a+b \right |=\left | b+c \right |=\left | c+a \right |=2\Rightarrow a=b=c.
Vậy ta có Q.E.D \blacksquare

Number Theory 22

(USA Team Selected Test 2003)
[Bài toán]: Tìm bộ ba số nguyên tố p,q,r thỏa mãn điều kiện: p\mid q^r+1,q\mid r^p+1,r\mid p^q+1
                                                                     Lời Giải
\blacktriangleright Bổ đề: Cho a,b,c\in \mathbb{P},c lẻ sao cho c\mid a^b+1 thì 2b\mid c-1;c\mid a^2-1
\rightarrow CM: Đặt d=\text{ord}_c(a)\qquad (d\in \mathbb{Z^+})
Ta có: c\mid a^b+1;c>2\Rightarrow a^b\equiv -1\pmod c,a^{2b}\equiv 1\pmod c
\Rightarrow d\mid 2b;d\mid b\Rightarrow d\in \{2;2b\}
+, Với d=2bd\mid c-1 (vì \text{ord}_c(a)\mid \phi(c)=c-1 do c\in \mathbb{P}) \Rightarrow 2b\mid c-1
+, Với d=2\Rightarrow a^2\equiv 1\pmod c\Rightarrow c\mid a^2-1
\blacktriangleright Áp dụng:
(*) Nếu có ít nhất 2 số bằng nhau, dễ dàng ta thấy điều vô lí!
(*) Không mất tính tổng quát, giả sử r<p<q
+, Nếu cả p,q,r đều lẻ. Ta có: q\mid r^p+1 \Rightarrow \begin{cases} 2p\mid q-1\\q\mid r^2-1\end{cases} \Rightarrow \begin{cases} q-1\geq 2p\\q\mid (r-1)(r+1)\end{cases}
Dễ dàng thấy trường hợp thứ nhất vô lí vì q<p
Ở trường hợp thứ hai, ta thấy \gcd(r-1;r+1)=2,p\in \mathbb{P}\Rightarrow q\mid \displaystyle{\frac{r-1}{2}} hoặc q\mid \displaystyle{\frac{r+1}{2}}q<\displaystyle{\frac{r-1}{2}<\frac{r+1}{2}}<r (trái với giả thiết nên trường hợp này loại)
+, Nếu trong 3 số có 1 số chẵn mà r<q<p\Rightarrow r=2
Ta có: r\mid p^q+1\Rightarrow \begin{cases}4=2q\mid r-1\\r\mid p^2-1\end{cases} p\mid q^r+1\Rightarrow \begin{cases} 2r\mid p-1\\p\mid q^2-1\end{cases} \Leftrightarrow p\mid 3\Rightarrow p=3
\Rightarrow r\mid 10p\in \mathbb{P}\Rightarrow r\in \{2;5\}r>2\Rightarrow r=5 (Thử lại thỏa mãn)
Vậy (p,q,r)=(2;3;5) và hoán vị thỏa mãn ycđb \blacksquare

 

Number Theory 20

(Junior Balkan MO 2000)
[Bài toán] Tìm n\in \mathbb{Z^+} sao cho n^2+3^n là số chính phương.
                                                           Lời Giải
Đặt n^2+3^n=a^2(1)(a\in \mathbb{Z^+})
\Leftrightarrow (a-n)(a+n)=3^n \Rightarrow\begin{cases}a-n=3^m\\ a+n=3^{n-m}\end{cases} (m\in \mathbb{Z^+})
Ta có: m<c-m\Leftrightarrow 2m<c\Leftrightarrow 2m+1\leq c
\blacktriangleright Xét n=2m+1 \Rightarrow \begin{cases}a-n=3^m\\a+n=3^{m+1}\end{cases}
\Rightarrow a+n=3(a-n)
\Leftrightarrow a=2n\Rightarrow n^2+3^n=4n^2
\Leftrightarrow 3^n=3n^2\Leftrightarrow 3^{n-1}=n^2 \Rightarrow n lẻ.
Đặt n=2k+1(k\in \mathbb{N}) \Rightarrow (3^k)^2=(2k+1)^2\Leftrightarrow 3^k=2k+1
Áp dụng BĐT Bernulli: 1+2k\leq (1+2)^k=3^k
\Rightarrow k\in \{0;1\}\Rightarrow n\in \{1;3\} (Thử lại thỏa mãn)
\blacktriangleright Xét n>2m+1\Rightarrow n\geq 2m+2
Ta có: 2n=3^{n-m}-3^m=9.3^{n-m-2}-3^m3^m\leq 3^{n-m-2}\Leftrightarrow n\geq 2m+2(\text{True})
Áp dụng bất đẳng thức Bernulli:
\Rightarrow 2n\geq 8.3^{n-m-2}=8.(1+2)^{n-m-2}\geq 8[1+2(n-m-2)]=16n-16m-24
\Leftrightarrow 7n\leq 8m+127n\geq 14m+14\Rightarrow 6m+2\leq 0 (vô lí)
Vậy \boxed{n\in \{1;3\}} thỏa mãn \blacksquare

Inequality 12

[Bài toán]: Cho a,b,c\geq 0 thỏa mãn a+b+c=2. CMR: (a^2+ab+b^2)(b^2+bc+c^2)(c^2+ca+a^2)\leq 3
                                                               Lời Giải
Không mất tính tổng quát, giả sử a\geq b\geq c\geq 0.
Đặt \displaystyle{t=\frac{a+b}{2};u=\frac{a-b}{2}(t,u\geq 0;t\leq 1)}
\rightarrow a^2+2ab+b^2=4t^2;a^2-2ab+b^2=4u^2
\blacktriangleright Ta phải CM: (a^2+ab+b^2)(b^2+bc+c^2)(c^2+ca+a^2)\leq 3t^2(t^2+tc+c^2)^2\qquad (*)
Ta có: +, a^2+ab+b^2=2t^2+2u^2+t^2-u^2=3t^2+u^2
+, (b^2+bc+c^2)(c^2+ca+a^2)
=c^4+a^2b^2+b^2c^2+c^2a^2+c^3(a+b)+abc(a+b+c)
=c^4+(t^2-u^2)^2+2c^2(t^2+u^2)+2tc^3+c(t^2-u^2)(c+2t)
=c^4+t^4-2t^2u^2+u^4+2t^2c^2+2u^2c^2+2tc^3+t^2c^2+2t^3c-c^2u^2-2tcu^2
=c^4+3t^2c^2+t^4+2tc(t^2+c^2)-u^2(2tc-c^2+2t^2-u^2)
Ta phải CM: \displaystyle{\frac{3t^2}{a^2+ab+b^2}-1\geq \frac{(b^2+bc+c^2)(c^2+ca+a^2)}{(t^2+tc+c^2)^2}-1}
\Leftrightarrow \displaystyle{\frac{3t^2-3t^2-u^2}{3t^2+u^2}\geq \frac{-u^2(2tc-c^2+2t^2-u^2)}{(t^2+tc+c^2)^2}}
\Leftrightarrow \displaystyle{\frac{1}{3t^2+u^2}\leq \frac{2tc-c^2+2t^2-u^2}{(t^2+tc+c^2)^2}}
\Leftrightarrow (2tc-c^2+2t^2-u^2)(3t^2+u^2)\geq (t^2+tc+c^2)^2
\Leftrightarrow 6t^3c+2tcu^2-3t^2c^2-c^2u^2+6t^4+2t^2u^2-3t^2u^2-u^4\geq t^4+t^2c^2+c^4+2t^3c+2tc^3+2t^2c^2
\Leftrightarrow 4t^3-2tc^3-6t^2c^2+5t^4-c^4\geq u^2c^2-2tc^2+t^2u^2+u^4=u^2(t-c)^2+u^4
Lại có:  5t^4+4t^3c-6t^2c^2+5t^4-c^4\geq 5t^4-5t^2c^2=5t^2(t-c)(t+c)
=5(t^3+t^2c)(t-c)\geq 5t^3(t-c)\geq 2(t-c)^4\geq u^2(t-c)^2+u^4\rightarrow (*) được CM.
\blacktriangleright Cuối cùng, ta phải CM: 3t^2(t^2+tc+c^2)^2\leq 3
Ta có: t=\displaystyle{\frac{a+b}{2}}\Rightarrow c=2-2t
\rightarrow t^2[t^2+t(2-2t)+(2-2t)^2]^2\leq 1\Leftrightarrow 3t^3-6t^2+4t-1\leq 0
\Leftrightarrow (3t^2-3t+1)(t-1)\leq 0\rightarrow \text{True}
Vậy bất đẳng thức ban đầu được CM, dấu bằng xảy ra \Leftrightarrow (a,b,c)=(1;1;0) và hoán vị \blacksquare

Number Theory 18

(Turkey Junior Balkan MO TST 2014)
Tìm bộ ba nghiệm nguyên dương (a,b,c) thỏa mãn phương trình (a^3+b)(b^3+a)=2^c \qquad (1)
                                                                 Lời Giải
Vì vai trò a,b như nhau nên giả sử a\geq b
\Rightarrow\begin{cases}a^3+b=2^m \\b^3+a=2^{c-m}\end{cases}\qquad (m\in \mathbb{Z^+};m<c)
\blacktriangleright Xét a=1\Rightarrow \begin{cases}1+b=2^m \\b^3+1=2^{c-m}\end{cases} \Leftrightarrow 2^m(b^2-b+1)=2^{c-m}
\Leftrightarrow b(b-1)+1=2^{c-2m}VT lẻ \Rightarrow VP lẻ \Rightarrow c-2m=0\Leftrightarrow c=2m \Rightarrow b(b-1)+1=1\Leftrightarrow b=1\Rightarrow c=2 (Thử lại thỏa mãn)
\blacktriangleright Xét a\geq 2. Xét biểu thức:
ab(ab-1)(ab+1)=a^3b^3-ab=b^3(a^3+b)-b(b^3+a)=b^3.2^m-b.2^{c-m}
=2^m(b^3-b.2^{c-2m})\Rightarrow 2^m\mid ab(ab-1)(ab+1) mà dễ thấy \gcd(ab,a^2b^2-1)=1 \Rightarrow 2 trường hợp sau:
(*) Trường hợp 2^m\mid ab\Rightarrow ab\geq 2^m\Leftrightarrow ab\geq a^3+b
\displaystyle{\Leftrightarrow b\geq \frac{a^3}{a-1}}
\displaystyle{\frac{a^3}{a-1}}> a\Leftrightarrow a(a^2-a+1)>0(\text{True})
\Rightarrow b>a (mâu thuẫn với giả sử ban đầu)
(*) Xét 2^m\mid (ab-1)(ab+1), mà ta thấy \gcd(ab-1,ab+1)=2 \Rightarrow 2 trường hợp:
+, Nếu \displaystyle{2^{m-1}\mid ab-1\Rightarrow \frac{a^3+b}{2}\mid ab-1\Rightarrow a^3+b\mid 2ab-2\Rightarrow \frac{2a^3b-2a^2}{a^3+b}}\in \mathbb{Z} \Rightarrow \displaystyle{\frac{2b(a^3+b)-2a^2-2b^2}{a^3+b}\in \mathbb{Z}\Rightarrow \frac{2a^2+2b^2}{a^3+b}}\in \mathbb{Z^+}
\Rightarrow 2a^2+2b^2\geq a^3+b\Leftrightarrow 2b^2-b\geq a^3-2a^2\geq b^3-2b^2
\Leftrightarrow b^3-4b^2+b\leq 0\Leftrightarrow b^2-4b+1\leq 0
\Leftrightarrow 2-\sqrt{3}\leq b\leq 2+\sqrt{3}\Rightarrow 1\leq b\leq 3(b\in \mathbb{Z^+})
2^{m-1}\mid ab-1\Rightarrow ab lẻ \Rightarrow b lẻ \Rightarrow b\in \{1;3\}
• Với b=1\Rightarrow (a,b)=(1;2)
• Với b=3\Rightarrow (1) \Leftrightarrow (a^3+3)(a+27)=2^c\Rightarrow (a,c)=(5;12)
+, Nếu 2^{m-1}\mid ab+1\Rightarrow \displaystyle{\frac{2a^3b+2b^2}{a^3+b}}\in \mathbb{Z^+}\Rightarrow \displaystyle{\frac{2a^2-2b^2}{a^3+b}}\in \mathbb{Z^+}\Rightarrow 2a^2-2b^2\geq a^3+b
\Leftrightarrow -2b^2-b\geq a^3-2a^2\geq b^3-2b^2\Leftrightarrow b^3+b\leq 0 (vô lí)
Vậy phương trình có nghiệm nguyên dương là: \boxed{(a,b,c)\in \{(1;1;2);(3;5;12);(5;3;12)\}}